The Flat Earth Society

Other Discussion Boards => Science & Alternative Science => Topic started by: Realestfake on December 06, 2023, 04:49:27 AM

Title: Do rockets push off the air?
Post by: Realestfake on December 06, 2023, 04:49:27 AM
I’d love to get discussion going on this. Do rockets move by pushing off the air?
It sure makes sense.
Title: Re: Do rockets push off the air?
Post by: Tom Bishop on December 06, 2023, 05:59:56 PM
They probably move though multiple mechanisms. They propel themselves through inertial mass ejection, and also push off of air when available near the exhaust port yes.
Title: Re: Do rockets push off the air?
Post by: Action80 on December 06, 2023, 08:50:54 PM
I’d love to get discussion going on this. Do rockets move by pushing off the air?
It sure makes sense.
Rockets eject a plume of gas from the bell nozzle. This allows them to move.

Jet engines do the same.
“After compression it was heated, augmented by additional burning fuel(reported in the press to be kerosene), and finally discharged from the aft vent in a monstrous jet of energy pushing against the atmosphere.”
(http://i.imgur.com/p3JANVi.jpg)

As long as either a jet or rocket engine can eject gas that is capable of forming a plume, then work (i.e., movement) can take place.

If a plume cannot form (i.e., no external pressure facilitating the formation of a plume), no work (i.e., movement) will take place.

Title: Re: Do rockets push off the air?
Post by: Realestfake on December 06, 2023, 10:21:35 PM
They probably move though multiple mechanisms. They propel themselves through inertial mass ejection, and also push off of air when available near the exhaust port yes.

Would intertial mass ejection work in a vacuum?
Title: Re: Do rockets push off the air?
Post by: Realestfake on December 06, 2023, 10:24:18 PM
As long as either a jet or rocket engine can eject gas that is capable of forming a plume, then work (i.e., movement) can take place.
So air resistance is the force that is moving rockets? Air resistance being the “pushing off the atmosphere”?
Title: Re: Do rockets push off the air?
Post by: markjo on December 06, 2023, 11:10:46 PM
Jet engines do the same.
“After compression it was heated, augmented by additional burning fuel(reported in the press to be kerosene), and finally discharged from the aft vent in a monstrous jet of energy pushing against the atmosphere.”
The atmoplane doesn't seem very solid to me.  Wouldn't that "monstrous jet of energy" just push the atmoplane out of its way, especially at higher altitudes where the air is much thinner?
Title: Re: Do rockets push off the air?
Post by: DuncanDoenitz on December 06, 2023, 11:27:36 PM
I began training as a Jet Engine Engineer in 1969; it was my career. 

I've no idea what the "plume" is that Action80 refers to in respect of a gas turbine engine.  They categorically do not rely on "resistance", or "pushing off the atmosphere", but please don't allow facts to get in the way of a good theory. 

Contrary to its title, "Popular Science" is not a professional scientific journal, it is a digest of scientific articles written for the layman.  The issue in question appears to date from the 1940s, and the article admits that much of the technology discussed is still secret.  The statement that thrust is obtained by a ("monstrous") jet of energy pushing against the atmosphere" is incorrect, as well as employing hyperbole. 

The diagram top-left of the article is false; there is no such component as a "carburation chamber (C)" in a gas turbine engine, nor does it illustrate the turbine. 

And Tom; "They probably move through multiple mechanisms"?  Glad we nailed that. 
Title: Re: Do rockets push off the air?
Post by: markjo on December 07, 2023, 03:42:19 AM
I've no idea what the "plume" is that Action80 refers to in respect of a gas turbine engine. 

My guess is that he's referring to this:
(https://qph.cf2.quoracdn.net/main-qimg-cdd217d203049232f8274df92d288f6b-lq)
Title: Re: Do rockets push off the air?
Post by: Action80 on December 07, 2023, 05:03:05 AM
As long as either a jet or rocket engine can eject gas that is capable of forming a plume, then work (i.e., movement) can take place.
So air resistance is the force that is moving rockets? Air resistance being the “pushing off the atmosphere”?
As long as a plume is capable of being formed (and this can only take place in a pressurized environment, external to the engine exhaust), then the craft can move.

I began training as a Jet Engine Engineer in 1969; it was my career. 

I've no idea what the "plume" is that Action80 refers to in respect of a gas turbine engine.  They categorically do not rely on "resistance", or "pushing off the atmosphere", but please don't allow facts to get in the way of a good theory.
Ah yes, please do not allow the facts I posted prevent you from straw-manning the living bejesus out of the post.

Pathetic.
Contrary to its title, "Popular Science" is not a professional scientific journal, it is a digest of scientific articles written for the layman.
True.
The issue in question appears to date from the 1940s, and the article admits that much of the technology discussed is still secret.
True.
The statement that thrust is obtained by a ("monstrous") jet of energy pushing against the atmosphere" is incorrect, as well as employing hyperbole.
The statement was true then, it is true now, and will remain true as long as jets or rockets remain in use. Nobody gives a good goddamn about your protestations otherwise. 
The diagram top-left of the article is false; there is no such component as a "carburation chamber (C)" in a gas turbine engine, nor does it illustrate the turbine.
Ignoring the fact it is a simplified diagram sourced from "Flight," magazine and not meant to be technical at all, I forget whether it was Heinkel or Messerschmitt, but fuel delivery in the early days did take place by using carburetors rather than injectors, so it does not qualify as "false."

Nobody gives a damn whether it illustrates the turbine or not.

Nobody gives a damn whether you think the word "monstrous," is hyperbole or not.

The jet engine and the rocket engine accomplish their work (i.e., movement) by ejecting a gas, which in turn forms a plume (which can only take place if there is an externally pressurized environment).

Title: Re: Do rockets push off the air?
Post by: Realestfake on December 07, 2023, 05:06:23 AM
Ah yes, please do not allow the facts I posted prevent you from straw-manning the living bejesus out of the post.

Pathetic.

Nobody gives a damn whether you think the word "monstrous," is hyperbole or not.

Why the negativity? You’re the first one to be disrespectful in this thread. That truly reflects on your character.

The jet engine and the rocket engine accomplish their work (i.e., movement) by ejecting a gas, which in turn forms a plume (which can only take place if there is an externally pressurized environment).
Interesting. So would ejecting a gas to move a rocket work in a vacuum?


Ill actually be more concise to what I’m getting at:
What does “pushing off of something” actually mean? Why should something move if it pushes against another object?
Title: Re: Do rockets push off the air?
Post by: Action80 on December 07, 2023, 05:37:25 AM
Ah yes, please do not allow the facts I posted prevent you from straw-manning the living bejesus out of the post.

Pathetic.

Nobody gives a damn whether you think the word "monstrous," is hyperbole or not.

Why the negativity? You’re the first one to be disrespectful in this thread. That truly reflects on your character.
Did you start a thread about character or how rockets push off air?

The jet engine and the rocket engine accomplish their work (i.e., movement) by ejecting a gas, which in turn forms a plume (which can only take place if there is an externally pressurized environment).
Interesting. So would ejecting a gas to move a rocket work in a vacuum?
No, a plume cannot form in an environment where there is no pressure.

Ill actually be more concise to what I’m getting at:
What does “pushing off of something” actually mean? Why should something move if it pushes against another object?
There is nothing more "concise," about this additional query.
Title: Re: Do rockets push off the air?
Post by: Realestfake on December 07, 2023, 05:52:48 AM
No, a plume cannot form in an environment where there is no pressure.

They are called plumes regardless of being in a vacuum or not. The actual shape doesn’t make it not a plume.

To cite this paper, for example: https://doi.org/10.1016/j.cja.2012.12.016

“In vacuum environment, the exhaust flow of the attitude control thrusters of satellites, spaceships, space stations and deep-space probes would expand freely and produce the plume. The vacuum plume would possibly cause undesirable contamination, aerodynamic force and thermal effects on the spacecraft, which would debase the capability of functional components and even make the flight missions fail. Therefore, for spacecraft designers the effects of the plume must be taken into account.”

I have no idea why you would make this claim.

(https://ars.els-cdn.com/content/image/1-s2.0-S1000936112000246-gr3.jpg)
Title: Re: Do rockets push off the air?
Post by: Action80 on December 07, 2023, 05:58:51 AM
No, a plume cannot form in an environment where there is no pressure.

They are called plumes regardless of being in a vacuum or not. The actual shape doesn’t make it not a plume.
A plume cannot form in a vacuum.

To cite this paper, for example: https://doi.org/10.1016/j.cja.2012.12.016

“In vacuum environment, the exhaust flow of the attitude control thrusters of satellites, spaceships, space stations and deep-space probes would expand freely and produce the plume. The vacuum plume would possibly cause undesirable contamination, aerodynamic force and thermal effects on the spacecraft, which would debase the capability of functional components and even make the flight missions fail. Therefore, for spacecraft designers the effects of the plume must be taken into account.”

I have no idea why you would make this claim.
First, your paper clearly states the altitudes are at "60–80 km..." which is not a "vacuum environment."

Second, rocket engines can and do indeed work in lower-pressure environments than jet engines.
Title: Re: Do rockets push off the air?
Post by: Realestfake on December 07, 2023, 06:07:44 AM
A plume cannot form in a vacuum.

It’s literally just the word for the gases that come out of a propulsive engine. If it makes any shape, that shape is called a plume.
Title: Re: Do rockets push off the air?
Post by: Action80 on December 07, 2023, 06:12:36 AM
A plume cannot form in a vacuum.

It’s literally just the word for the gases that come out of a propulsive engine. If it makes any shape, that shape is called a plume.
There is a shape in both of your pictures, of course. One is more defined. but again, your paper is specifically writing, ""60-80 km...". Not an outer space vacuum environment.
Title: Re: Do rockets push off the air?
Post by: Realestfake on December 07, 2023, 06:16:56 AM
There is a shape in both of your pictures, of course. One is more defined. but again, your paper is specifically writing, ""60-80 km...". Not an outer space vacuum environment.

What’s the altitude where the plume stops being a plume and why? Why is the shape of exhaust arbitrarily no longer called a plume to you if you go higher?
And I say “to you” because I linked the article in the first place to cite usage of the word “plume” relating to, quote: “the exhaust flow of the attitude control thrusters of satellites, spaceships, space stations and deep-space probes would expand freely and produce the plume.”
Title: Re: Do rockets push off the air?
Post by: Action80 on December 07, 2023, 06:43:36 AM
There is a shape in both of your pictures, of course. One is more defined. but again, your paper is specifically writing, ""60-80 km...". Not an outer space vacuum environment.

What’s the altitude where the plume stops being a plume and why? Why is the shape of exhaust arbitrarily no longer called a plume to you if you go higher?
And I say “to you” because I linked the article in the first place to cite usage of the word “plume” relating to, quote: “the exhaust flow of the attitude control thrusters of satellites, spaceships, space stations and deep-space probes would expand freely and produce the plume.”
"Deep space," is your first clue.

"60-80 km..." does not constitute "deep space..."

It seems your article is inserting contradictory adjectives.
Title: Re: Do rockets push off the air?
Post by: Realestfake on December 07, 2023, 06:56:52 AM
What’s the altitude where the plume stops being a plume and why? Why is the shape of exhaust arbitrarily no longer called a plume to you if you go higher?

It seems your article is inserting contradictory adjectives.

The article is talking about different things at different times. These aren’t contradictory.
What aren’t you getting about calling a plume from a deep space probe a plume? Is it because they, in the same article, describe the simulated altitude (60-80km) of their low-pressure chamber? Does that undo them calling a deep space probe’s plume a plume? Is that the road you’re going down?


“In the space vacuum the exhaust gases form a large free jet, called a plume, which can impinge on neighbouring surfaces.”
https://doi.org/10.1016/0376-0421(91)90008-R
Title: Re: Do rockets push off the air?
Post by: Action80 on December 07, 2023, 07:18:13 AM
What’s the altitude where the plume stops being a plume and why? Why is the shape of exhaust arbitrarily no longer called a plume to you if you go higher?

It seems your article is inserting contradictory adjectives.

The article is talking about different things at different times. These aren’t contradictory.
What aren’t you getting about calling a plume from a deep space probe a plume? Is it because they, in the same article, describe the simulated altitude (60-80km) of their low-pressure chamber? Does that undo them calling a deep space probe’s plume a plume? Is that the road you’re going down?


“In the space vacuum the exhaust gases form a large free jet, called a plume, which can impinge on neighbouring surfaces.”
https://doi.org/10.1016/0376-0421(91)90008-R
Listen, if you don't understand the words, "deep space," are not synonymous with "60-80 km," then maybe you should check another forum instead.
Title: Re: Do rockets push off the air?
Post by: Realestfake on December 07, 2023, 07:27:46 AM
”In the space vacuum the exhaust gases form a large free jet, called a plume, which can impinge on neighbouring surfaces.”
https://doi.org/10.1016/0376-0421(91)90008-R
Title: Re: Do rockets push off the air?
Post by: Action80 on December 07, 2023, 07:36:47 AM
”In the space vacuum the exhaust gases form a large free jet, called a plume, which can impinge on neighbouring surfaces.”
https://doi.org/10.1016/0376-0421(91)90008-R
Again, switching between the terms "space," and "deep space," is kinda funny.

Rockets can work at altitudes of 60-80 km.
Title: Re: Do rockets push off the air?
Post by: Realestfake on December 07, 2023, 07:40:09 AM
”In the space vacuum the exhaust gases form a large free jet, called a plume, which can impinge on neighbouring surfaces.”
https://doi.org/10.1016/0376-0421(91)90008-R
Again, switching between the terms "space," and "deep space," is kinda funny.

Rockets can work at altitudes of 60-80 km.

This is a different paper. It makes no mention of 60-80 km. Not surprised you didn’t actually read any of it.
You know the terms “space” and “deep space” aren’t just randomly used interchangeably right?
Title: Re: Do rockets push off the air?
Post by: Action80 on December 07, 2023, 07:48:30 AM
This is a different paper. It makes no mention of 60-80 km. Not surprised you didn’t actually read any of it.
You know the terms “space” and “deep space” aren’t just randomly used interchangeably right?
There was no "paper," just an abstract and a list of references, so you didn't, "read it," either.

And I agree the terms, "aren’t just randomly used interchangeably," so it would be beneficial for everyone if you stopped doing just that.
Title: Re: Do rockets push off the air?
Post by: Realestfake on December 07, 2023, 04:12:19 PM
And I agree the terms, "aren’t just randomly used interchangeably," so it would be beneficial for everyone if you stopped doing just that.

I… didn’t? No amount of squirming your way out of it (“heh, pick between space and deep space!”, “erm… but they said 60-80k in the same article when talking about something else…”, “erm… I will ignore the contents of what was sent because IT’S JUST REFERENCES”) changes the fact that scientists call the jet of gases coming out a rocket in space a plume. Your willful avoidance of this is just not a good look on your part  :(


”In the space vacuum the exhaust gases form a large free jet, called a plume, which can impinge on neighbouring surfaces.”
https://doi.org/10.1016/0376-0421(91)90008-R

Anything else you have said since me bringing this up has been painfully obvious avoidance of the topic. You have arbitrarily established that plumes cannot form without pressure. There is literally nothing anywhere that demonstrates or claims this. You made it too easy.
Title: Re: Do rockets push off the air?
Post by: Action80 on December 07, 2023, 05:02:52 PM
And I agree the terms, "aren’t just randomly used interchangeably," so it would be beneficial for everyone if you stopped doing just that.

I… didn’t?
Yeah, you did...
[/quote]
deep-space .”
space
Anything else you have said since me bringing this up has been painfully obvious avoidance of the topic. You have arbitrarily established that plumes cannot form without pressure. There is literally nothing anywhere that demonstrates or claims this. You made it too easy.
I have not avoided the topic at all.

I already stated how rockets move.

A gas plume from a rocket cannot form in an environment absent of external pressure.

That is the fact of the matter and nothing you have provided states otherwise, aside from the usual gaslighting bs practiced by a bunch of know-nothings trying to pass themselves off as expurtts.

They use a generic term like "space," just like you do, deliberately misleading people into picturing something like a scene from Buck Rodgers, when in fact a rocket will perform work in "space," as long that space has an adequate amount of pressure to allow a plume to form when the rocket ejects the self-contained gas.

A vacuum in "outer space" such as reported by RE-adherents, does not possess enough environmental pressure. 
Title: Re: Do rockets push off the air?
Post by: RonJ on December 07, 2023, 05:35:51 PM
The ‘plumb’ in space doesn’t have anything to do with the production of a force by the rocket engine.  A rocket contains fuel that has a mass, and an amount of dormant energy.  When combustion occurs, the dormant energy is released, and that energy effectively accelerates the mass of the fuel.  The combusted fuel is accelerating out the back of the rocket.  When you apply Newton’s law you have an equal and opposite amount of force (F = MA) applied on the rocket in the direction opposite to the direction of the exiting combusted fuel.  It doesn’t matter if the rocket’s environment is in air or a vacuum.  Newton’s law applies equally in either environment. 
Title: Re: Do rockets push off the air?
Post by: DuncanDoenitz on December 07, 2023, 05:41:55 PM
Whether a plume can form in a vacuum or not, can you expand on its relevance to propulsion?  Your opening hypothesis is that a plume somehow imparts movement to the rocket, but you have not explained how this happens; what is the science behind this? 
Title: Re: Do rockets push off the air?
Post by: Realestfake on December 07, 2023, 05:47:50 PM
I already stated how rockets move.

You stated how you think rockets move, as did Tom Bishop. Nothing of value was provided as evidence or reasoning.
If I throw a bowling ball while standing on a skateboard, did air resistance move me in the opposite direction?
Title: Re: Do rockets push off the air?
Post by: Realestfake on December 07, 2023, 06:22:49 PM
And I agree the terms, "aren’t just randomly used interchangeably," so it would be beneficial for everyone if you stopped doing just that.

I’m sorry Action, but you’re just fundamentally unprepared for this debate. You were unable to discern the terminology of “space” vs “deep space” and inappropriately attributed to a mistake on MY end.

To quote the article: “satellites, spaceships, space stations and deep-space probes”

This is absolutely correct usage because probes are in fact designed for deep space, while space stations are in LEO. You saying I’m using them “interchangeably” demonstrates your own lack of understanding  :(
I recognize you have nothing to add and honestly? That’s okay. You have time to learn still.
Title: Re: Do rockets push off the air?
Post by: Action80 on December 07, 2023, 07:08:14 PM
Ronj is going on about a "plumb," he must have pulled out with his thumb over in a corner somewhere. Conveniently forgetting that no "plumb," assigns a big fat 0 to the A in the equation. I don't care if you multiply 1,000,000 M's to the big fat 0 of A, F will end up as a big fat 0.

Donutz is still asking how jets move when he is supposedly a former jet engine mechanic (by the way my nephew is currently employed by United Airlines, having served over 10 years as a former Air Force jet engine mechanic). Jets too, form a plume in the pressurized atmoplane, allowing them to move. The only difference in accomplishing their operation of movement is jets require an intake of external oxygen to achieve combustion (open system), whereas rockets do not have air intakes and have a self-contained material allowing combustion to take place when it is mixed with the fuel.

And faker is going on about who is ill-prepared, when his lack of reasoning has been so thoroughly discredited on this site it is laughable.

You are the one, faker, who was posting the bounce-around terms, clearly subscribing to them, just willy-nilly all over the place.

Really laughable.

Typical.

Rockets work in a pressurized environment where they can form a plume.

No external pressure outside the rocket?

No plume.

No plume?

No movement.

The end.


Title: Re: Do rockets push off the air?
Post by: Realestfake on December 07, 2023, 08:04:27 PM
No external pressure outside the rocket?

No plume.

No plume?

No movement.

The end.

If I throw a bowling ball while standing on a skateboard, did I move backwards because of air resistance?

I’m going to make this incredibly simple.
You want to use the “pushing off an atmosphere” idea.
Okay.
Imagine the inside of an engine. The explosive power of the combustion pushes against the inside of the engine opposite of the plume, moving the rocket. No part of that process required an atmosphere.
The rocket is pushing against something inside itself.
Title: Re: Do rockets push off the air?
Post by: RonJ on December 07, 2023, 08:23:02 PM
You put some rocket fuel inside a rocket and ignite the fuel to release a quantity of energy.  The fuel is transformed from a solid (or liquid) to a very hot expanding gas.  The gas is confined inside the rocket ignition chamber with only one way out.  Pressure builds up and the very hot gas exits thru the engine’s nozzle that helps to accelerate the gas.  The hot exiting gas’s mass will be equal to the mass of the fuel that was burned.  Now you have the mass(M) in the equation.  The hot gas exiting the rocket engine is traveling at a high speed so, since it was almost zero velocity at the start, has accelerated greatly while traveling through the nozzle.  Now you have the (A) acceleration part.  Since the force is equal to the mass times the acceleration, you have a force opposite the direction of the rockets exiting gas.  There’s no air mentioned in the equation anywhere. All that is needed is an accelerating mass.   


If anything, the surrounding air will only slow the rocket’s acceleration.  Any force produced by the air pushing the rocket up will be balanced out by the force of the air pushing back on the nose of the rocket.  If the surface of the earth was a vacuum the acceleration of the rocket would be greater, all things being equal. 
Title: Re: Do rockets push off the air?
Post by: Dr Van Nostrand on December 07, 2023, 08:24:00 PM
Donutz is still asking how jets move when he is supposedly a former jet engine mechanic (by the way my nephew is currently employed by United Airlines, having served over 10 years as a former Air Force jet engine mechanic). Jets too, form a plume in the pressurized atmoplane, allowing them to move.

lol   so... Does your nephew agree with you that rockets only work within the confines of an atmosphere? Did he learn that is his Air Force training? You two could challenge everything we know about physics.

If I throw a bowling ball while standing on a skateboard, did I move backwards because of air resistance?

This was all thoroughly debated in another thread and every example was proposed. An astronaut floating in space fires a shotgun, a catapult launching a rock, a dude in a vacuum sits on a hand grenade and doesn't move when it goes off... Apparently, our understanding of physics is all part of some liberal hoax cabal or something. Action80 and his nephew are the only ones who know the truth. You'd think that with such awesome knowledge that they'd be rich and famous engineers making $250,000 USD a year.
Title: Re: Do rockets push off the air?
Post by: Action80 on December 07, 2023, 08:26:30 PM
No external pressure outside the rocket?

No plume.

No plume?

No movement.

The end.

If I throw a bowling ball while standing on a skateboard, did I move backwards because of air resistance?
Using an inappropriate analogy isn't going to help you.
I’m going to make this incredibly simple.
You want to use the “pushing off an atmosphere” idea.
Okay.
Imagine the inside of an engine. The explosive power of the combustion pushes against the inside of the engine opposite of the plume, moving the rocket. No part of that process required an atmosphere.
The rocket is pushing against something inside itself.
No concept of a force pair exists in your fake and false description.

Combustion takes place separately and distinctly from the exhaust process.

Your posts are nonsensical and reek of desperation.

Hold onto your folly.

Leave reality to the sane.
Title: Re: Do rockets push off the air?
Post by: Action80 on December 07, 2023, 08:29:55 PM
An astronaut floating in space fires a shotgun, a catapult launching a rock, a dude in a vacuum sits on a hand grenade and doesn't move when it goes off...
I see where the faker gets his nonsensical and fake analogies.
Title: Re: Do rockets push off the air?
Post by: Realestfake on December 07, 2023, 08:36:55 PM
No concept of a force pair exists in your fake and false description.

Combustion takes place separately and distinctly from the exhaust process.

Your posts are nonsensical and reek of desperation.

Hold onto your folly.

Leave reality to the sane.

Explain in detail why it is “fake and false”. The rocket is being pushed against by an internal combustion (which results in gas being accelerated outwards). I really try to not debate with beginner-levels but I do want to help.


”In the space vacuum the exhaust gases form a large free jet, called a plume, which can impinge on neighbouring surfaces.”
https://doi.org/10.1016/0376-0421(91)90008-R
No, a plume cannot form in an environment where there is no pressure.

You not agreeing what a “plume” is against the rest of the world is literally nobody’s problem but yours. Lol.
Title: Re: Do rockets push off the air?
Post by: DuncanDoenitz on December 07, 2023, 08:39:26 PM
@Action. First of all respect to your nephew; serving his country in the military and then transferring those learned skills to the airline industry.  Similar career path to mine, though in the UK. 

Following up on Dr v-N, I wonder if you have actually discussed jet engine theory with your nephew, or whether you are just throwing in random relatives in the hope that it will lend your argument some kudos.  My sister is a nurse, but that wouldn't reinforce any argument I might make about Covid. 

And I don't like labouring a point, but you still haven't explained how the presence of a plume lends thrust to the jet/rocket. 
Title: Re: Do rockets push off the air?
Post by: Action80 on December 07, 2023, 09:14:58 PM
No concept of a force pair exists in your fake and false description.

Combustion takes place separately and distinctly from the exhaust process.

Your posts are nonsensical and reek of desperation.

Hold onto your folly.

Leave reality to the sane.

Explain in detail why it is “fake and false”. The rocket is being pushed against by an internal combustion (which results in gas being accelerated outwards). I really try to not debate with beginner-levels but I do want to help.
Jesus, you double down to claim it is only an internal combustion absent any exhaust resulting in movement.

Unbelievable!


I will spare you the rest here, because you are simply clueless.
”In the space vacuum the exhaust gases form a large free jet, called a plume, which can impinge on neighbouring surfaces.”
https://doi.org/10.1016/0376-0421(91)90008-R
No, a plume cannot form in an environment where there is no pressure.

You not agreeing what a “plume” is against the rest of the world is literally nobody’s problem but yours. Lol.
I agree what a plume is and have already stated as such.

Your LOL is simply you laughing at your own nonsense.

I am not the one with a problem, you are.
Title: Re: Do rockets push off the air?
Post by: Realestfake on December 07, 2023, 09:18:24 PM
Jesus, you double down to claim it is only an internal combustion absent any exhaust resulting in movement.

…what? Please reread what was said.
Why does a ballerina speed up when they pull their arms in?
Title: Re: Do rockets push off the air?
Post by: Action80 on December 07, 2023, 09:18:47 PM
@Action. First of all respect to your nephew; serving his country in the military and then transferring those learned skills to the airline industry.  Similar career path to mine, though in the UK. 

Following up on Dr v-N, I wonder if you have actually discussed jet engine theory with your nephew, or whether you are just throwing in random relatives in the hope that it will lend your argument some kudos.  My sister is a nurse, but that wouldn't reinforce any argument I might make about Covid. 

And I don't like labouring a point, but you still haven't explained how the presence of a plume lends thrust to the jet/rocket.
Yes, I have and he understands that a pressurized environment must exist for a plume to form.

The plume is like any appendage and is what allows the rocket or jet to push off the atmoplane.

No defined exhaust (plume)... no movement.
Title: Re: Do rockets push off the air?
Post by: Action80 on December 07, 2023, 09:20:33 PM
Jesus, you double down to claim it is only an internal combustion absent any exhaust resulting in movement.

…what? Please reread what was said.
Why does a ballerina speed up when they pull their arms in?
Do everyone a favor, okay?

Go peddle your nonsensical crapola elsewhere.

I am done with your dissimilar anologies.
Title: Re: Do rockets push off the air?
Post by: Realestfake on December 07, 2023, 09:24:14 PM
The plume is like any appendage and is what allows the rocket or jet to push off the atmoplane.

No defined exhaust (plume)... no movement.

Yeahh you’re going to need some supporting evidence or reasoning. Your claim is wildly inconsistent with basic observation.
Title: Re: Do rockets push off the air?
Post by: Realestfake on December 07, 2023, 09:25:32 PM
Do everyone a favor, okay?

Go peddle your nonsensical crapola elsewhere.

I am done with your dissimilar anologies.

Okay. Do you believe in conservation of angular momentum? If not, explain why (according to Action80ian physics) a ballerina speeds up when they pull their arms in.
I don’t know which part was nonsensical. Please be specific  :)
Or at the very least - try to keep up!
Title: Re: Do rockets push off the air?
Post by: Dr Van Nostrand on December 07, 2023, 09:59:05 PM
Yes, I have and he understands that a pressurized environment must exist for a plume to form.

Do his superiors and the engineers that designed the equipment he works on understand it?
If he were to ask the people who trained him about rockets and vacuums what would they say?

Are you smarter than them or are they lying to hide the truth?
Title: Re: Do rockets push off the air?
Post by: Action80 on December 07, 2023, 10:03:24 PM
Do everyone a favor, okay?

Go peddle your nonsensical crapola elsewhere.

I am done with your dissimilar anologies.

Okay. Do you believe in conservation of angular momentum? If not, explain why (according to Action80ian physics) a ballerina speeds up when they pull their arms in.
I don’t know which part was nonsensical. Please be specific  :)
Or at the very least - try to keep up!
What "angular momentum?" A rocket is pointed toward a direction during the propulsion phase (i.e., linear momentum).  If a rocket, taking off in a pressurized environment, enters a non-pressurized environment, it will continue to move until an equal opposite force acts upon it. But its movement will soon cease because the plume can no longer maintain its integrity.

Take any one of your analogies and feel free to apply the adjective "nonsense."

Is that specific enough for you? I hope so because it is true.

Very true.
Title: Re: Do rockets push off the air?
Post by: Action80 on December 07, 2023, 10:05:33 PM
Yes, I have and he understands that a pressurized environment must exist for a plume to form.

Do his superiors and the engineers that designed the equipment he works on understand it?
Of course they do.

If he were to ask the people who trained him about rockets and vacuums what would they say?
Pretty much the same thing I am.

Are you smarter than them or are they lying to hide the truth?
Lying about what?
Title: Re: Do rockets push off the air?
Post by: Realestfake on December 07, 2023, 10:41:50 PM
What "angular momentum?"

I’m not talking about rockets in this case. I’m asking simply if you believe the concept of conservation of angular momentum to be correct. Out of curiosity. Literally a yes or no question.
Title: Re: Do rockets push off the air?
Post by: DuncanDoenitz on December 07, 2023, 11:20:13 PM
Yes, I have and he understands that a pressurized environment must exist for a plume to form.

Do his superiors and the engineers that designed the equipment he works on understand it?
Of course they do.

If he were to ask the people who trained him about rockets and vacuums what would they say?
Pretty much the same thing I am.

Are you smarter than them or are they lying to hide the truth?
Lying about what?


You have an admirable confidence that:
a.   You have understood what your nephew means, and vice versa.
b.   You nephew has fully understood what he has been taught. 
c.   The jet engine designers, engineers and technicians are also in accordance with you. 

However;

http://www.valentiniweb.com/piermo/meccanica/mat/Rolls%20Royce%20-%20The%20Jet%20Engine.pdf

This is a link to a pdf version of a go-to publication in the UK, and also much of the English-speaking world.  It is called "The Jet Engine" (ISBN 0 902121 2 35) and its published by Rolls Royce, who know a couple of things about them.  The book is standard reading for anyone in the UK who is embarking on a career in aircraft engines.  It outlines the principles of theory, design, construction and maintnenane, and not just of Rolls Royce products. 

I draw your attention to Part 1 "Basic mechanics", page 2, Paras 6 thru 9:

6. Jet propulsion is a practical application of Sir
Isaac Newton's third law of motion which states that,
'for every force acting on a body there is an opposite
and equal reaction'. For aircraft propulsion, the 'body'
is atmospheric air that is caused to accelerate as it
passes through the engine. The force required to
give this acceleration has an equal effect in the
opposite direction acting on the apparatus producing
the acceleration. A jet engine produces thrust in a
similar way to the engine/propeller combination. Both
propel the aircraft by thrusting a large weight of air
backwards (fig. 1-3), one in the form of a large air
slipstream at comparatively low speed and the other
in the form of a jet of gas at very high speed.
7. This same principle of reaction occurs in all forms
of movement and has been usefully applied in many
ways. The earliest known example of jet reaction is
that of Hero's engine (fig. 1-4) produced as a toy in
120 B.C. This toy showed how the momentum of
steam issuing from a number of jets could impart an
equal and opposite reaction to the jets themselves,
thus causing the engine to revolve.
8. The familiar whirling garden sprinkler (fig. 1-5) is
a more practical example of this principle, for the
mechanism rotates by virtue of the reaction to the
water jets. The high pressure jets of modern firefighting equipment are an example of 'jet reaction',
for often, due to the reaction of the water jet, the hose
cannot be held or controlled by one fireman. Perhaps
the simplest illustration of this principle is afforded by
the carnival balloon which, when the air or gas is
released, rushes rapidly away in the direction
opposite to the jet.
9. Jet reaction is definitely an internal phenomenon
and does not, as is frequently assumed, result from
the pressure of the jet on the atmosphere.
In fact, the
jet propulsion engine, whether rocket, athodyd, or
turbo-jet, is a piece of apparatus designed to
accelerate a stream of air or gas and to expel it at
high velocity. There are, of course, a number of ways .....

I can find no mention of "plume" in the book, but be my guest.  Perhaps you could discuss this further with your nephew. 

Edit; my Bold, btw.
Title: Re: Do rockets push off the air?
Post by: markjo on December 07, 2023, 11:37:02 PM
What "angular momentum?"

I’m not talking about rockets in this case. I’m asking simply if you believe the concept of conservation of angular momentum to be correct. Out of curiosity. Literally a yes or no question.
You should have asked about conservation of momentum in general rather than angular momentum in particular.

I'm still curious about what quality of an exhaust plume allows it to push off a medium less dense than itself.
Title: Re: Do rockets push off the air?
Post by: Dr Van Nostrand on December 07, 2023, 11:48:37 PM
Are you smarter than them or are they lying to hide the truth?
Lying about what?
You know,
Rockets going to the moon.... satellites 20,000 miles up and all that.
Title: Re: Do rockets push off the air?
Post by: Realestfake on December 07, 2023, 11:51:09 PM
What "angular momentum?"

I’m not talking about rockets in this case. I’m asking simply if you believe the concept of conservation of angular momentum to be correct. Out of curiosity. Literally a yes or no question.
You should have asked about conservation of momentum in general rather than angular momentum in particular.

I'm still curious about what quality of an exhaust plume allows it to push off a medium less dense than itself.

I agree. I’m just curious on whether he accepts a “subcategory” as an explanation for some things but not other things.
Title: Re: Do rockets push off the air?
Post by: Action80 on December 08, 2023, 05:46:52 AM
9. Jet reaction is definitely an internal phenomenon
and does not, as is frequently assumed, result from
the pressure of the jet on the atmosphere.
I

I can find no mention of "plume" in the book, but be my guest.  Perhaps you could discuss this further with your nephew. 

Edit; my Bold, btw.
I am only going to concern myself with this portion.

So, all internal...

And yet all the arrows in Figure 1-1 show the exhaust traveling to to the rear.

If it truly was all internal, then the thrust would be traveling to the front, like some other jokers like to claim here.

It doesn't matter what your source claims, there is a plume related to all jets and rockets (i.e., we see what is typically called a contrail), and that plume reacts with the pressurized external environment to form a force pair, which results in movement. No force pair, no movement.
Title: Re: Do rockets push off the air?
Post by: Realestfake on December 08, 2023, 05:56:25 AM
It doesn't matter what your source claims, there is a plume related to all jets and rockets (i.e., we see what is typically called a contrail), and that plume reacts with the pressurized external environment to form a force pair, which results in movement. No force pair, no movement.

A plume is a jet of plasma. A contrail is condensed water. What does a contrail have to do with anything?
Also, the “force pair” is the gas pushing against the rocket and the rocket itself. Hope this helps!
Title: Re: Do rockets push off the air?
Post by: Action80 on December 08, 2023, 07:38:50 AM
It doesn't matter what your source claims, there is a plume related to all jets and rockets (i.e., we see what is typically called a contrail), and that plume reacts with the pressurized external environment to form a force pair, which results in movement. No force pair, no movement.

A plume is a jet of plasma. A contrail is condensed water. What does a contrail have to do with anything?
Also, the “force pair” is the gas pushing against the rocket and the rocket itself. Hope this helps!
The exhausted gas is part of the rocket. A rocket is a closed system.

No force pair.
Title: Re: Do rockets push off the air?
Post by: DuncanDoenitz on December 08, 2023, 08:29:26 AM

It doesn't matter what your source claims, there is a plume related to all jets and rockets (i.e., we see what is typically called a contrail), and that plume reacts with the pressurized external environment to form a force pair, which results in movement. No force pair, no movement.

The "source" which designs, develops and manufactures jet engines, refuted by Action80's superior insight.  And possibly his nephew. 
Title: Re: Do rockets push off the air?
Post by: Action80 on December 08, 2023, 09:12:45 AM

It doesn't matter what your source claims, there is a plume related to all jets and rockets (i.e., we see what is typically called a contrail), and that plume reacts with the pressurized external environment to form a force pair, which results in movement. No force pair, no movement.

The "source" which designs, develops and manufactures jet engines, refuted by Action80's superior insight.  And possibly his nephew.
If your source claims there is no force pair, then it doesn't fucking matter what anybody says, that source is fucking wrong.

Jesus, how do you think you are going to get away with posting bullshit and somebody is not going to call you out for it?

A force pair is needed.

That is plain, pure simple physics (to quote the penguin). I cannot help you cannot read or understand what your source is claiming.
Title: Re: Do rockets push off the air?
Post by: DuncanDoenitz on December 08, 2023, 09:36:00 AM
Exhaust gas accelerates right.  Reaction applies a force left.  Force pair.  Can you specify where the RR Book denies this? 

And going back a couple of posts, can you clarify you meant Fig 1.5?  That's a garden sprinkler. 
Title: Re: Do rockets push off the air?
Post by: Action80 on December 08, 2023, 12:50:04 PM
Exhaust gas accelerates right.  Reaction applies a force left.  Force pair.  Can you specify where the RR Book denies this? 

And going back a couple of posts, can you clarify you meant Fig 1.5?  That's a garden sprinkler.
Figure 1-1, corrected.

The exhaust is part of the jet or rocket. You must have one thing reacting with an entirely separate thing, not just part of itself.
Title: Re: Do rockets push off the air?
Post by: Dr Van Nostrand on December 08, 2023, 01:48:41 PM
Exhaust gas accelerates right.  Reaction applies a force left.  Force pair.  Can you specify where the RR Book denies this? 

And going back a couple of posts, can you clarify you meant Fig 1.5?  That's a garden sprinkler.
Figure 1-1, corrected.

The exhaust is part of the jet or rocket. You must have one thing reacting with an entirely separate thing, not just part of itself.

You and your nephew have some pretty big balls to stand up in public and say that Newton's 3rd law of physics isn't real. I'm sure you must have some amazing proof that will blow away centuries of science. Do you have any evidence that doesn't come from your own mind or from YouTube?
Title: Re: Do rockets push off the air?
Post by: DuncanDoenitz on December 08, 2023, 02:06:10 PM


So, all internal...

And yet all the arrows in Figure 1-1 show the exhaust traveling to to the rear.

If it truly was all internal, then the thrust would be traveling to the front, like some other jokers like to claim here.



Got the correction; thanks.  This is a simplified diagram of the gas flow through the engine; it does not illustrate thrust.  It shows air being inducted from the left, compressed and impelled centrifugally by the compressor, diffused and entering the combustion chamber (to the right), passing though the turbine and exiting (to the right).  (Incidentally, the fact that the intake is to the left is just a convenience.  Many engines draw their air from all around, it doesn't matter.  The only important vector is that exhaust goes right, reactive thrust goes left). 

It is a simplified diagram is explaining the gas path.  To the target audience, the fact that thrust acts to the left does not require explanation.  Why would any of the arrows point left? 

An equivalent diagram for a road vehicle might show the engine, pistons, transmission and wheels going round.  The fact that the wheels try to push the road backwards does not need to be explained. 

Title: Re: Do rockets push off the air?
Post by: DuncanDoenitz on December 08, 2023, 02:42:06 PM
Whilst I echo Dr V-N's sentiments (and call me Mr Cynical) I am curious to what extent A80's nephew is on-board with this idea.  ("My uncle said what"?).  I mean, can we get an outline of how the conversation went?  Did you come up with the theory, or did you get it from  him?  When did you last discuss it?  After all, we only have your assertion that he is in agreement. 

As for General Electric and Pratt & Whitney being on the same page, without any references, this adds a whole 'nother stage of incredulity. 
Title: Re: Do rockets push off the air?
Post by: Action80 on December 08, 2023, 05:13:20 PM
You and your nephew have some pretty big balls to stand up in public and say that Newton's 3rd law of physics isn't real. I'm sure you must have some amazing proof that will blow away centuries of science. Do you have any evidence that doesn't come from your own mind or from YouTube?
You have no balls to stand up say that I have denied the third law of Newton. Equal and opposite reaction. Gas goes out one way, and the rocket or jet travels the other.

I am sure the only proof you have ever come up with in your life has some sort of warning label affixed to it warning about use during pregancy.
Title: Re: Do rockets push off the air?
Post by: Action80 on December 08, 2023, 05:21:27 PM


So, all internal...

And yet all the arrows in Figure 1-1 show the exhaust traveling to to the rear.

If it truly was all internal, then the thrust would be traveling to the front, like some other jokers like to claim here.



Got the correction; thanks.  This is a simplified diagram of the gas flow through the engine; it does not illustrate thrust.
Ah, yes...the good ole "ignore what your lying fucking eyes are looking at!" argument.
It shows air being inducted from the left, compressed and impelled centrifugally by the compressor, diffused and entering the combustion chamber (to the right), passing though the turbine and exiting (to the right).  (Incidentally, the fact that the intake is to the left is just a convenience.  Many engines draw their air from all around, it doesn't matter.  The only important vector is that exhaust goes right, reactive thrust goes left).
Exhaust gas generates the thrust, period. No exhaust gas, no thrust. Exhaust gas generates a plume.
It is a simplified diagram is explaining the gas path.  To the target audience, the fact that thrust acts to the left does not require explanation.  Why would any of the arrows point left?

Because if you read the posts of the asshat train of various posters here, they are gaslighting the shit out of the issue, claiming the exhaust gas is pushing somewhere to the front of the rocket within the combustion chamber, quite similar to what you are trying to now claim happens with gas turbine engines.
 

An equivalent diagram for a road vehicle might show the engine, pistons, transmission and wheels going round.  The fact that the wheels try to push the road backwards does not need to be explained.
Nobody gives a damn about your fake analogies and comparisons.
Title: Re: Do rockets push off the air?
Post by: Realestfake on December 08, 2023, 05:25:58 PM
You’re holding onto someone in a vacuum.
You push off each other. According to Action80, only one of you should move (the gas should move but not the rocket).
After all, before you both push you’re a “closed system”. It’s almost as if, when you push off the other person, the le momentum is… le conserved.

Rockets are, in fact, observed to gain efficiency at higher altitudes with less air resistance (varying slightly with the engine’s specified job)
Title: Re: Do rockets push off the air?
Post by: RonJ on December 08, 2023, 06:09:57 PM
General Electric gas turbine engines work on all the normal principles of physics you are taught in college engineering school.  I don’t need to consult with a relative of any kind as I am, personally, a federally certified gas turbine engineer.  There are some gas turbine powered, government owned, military ships in service that I have personally worked aboard many times.   A gas will always flow from a higher pressure towards a lower pressure.  Figure 1-2 on page 2 would be a more relevant diagram illustrating how a gas turbine works.  The compressor blades are spinning, and this accelerates the air.  This air mass acceleration is a source of the turbine engine’s forward thrust.  Where does the compressor get its power to accelerate the air?  You can see it’s via the shaft connecting the compressor with the turbine.  The turbine is powered by the released energy of the burning fuel inside the combustion chamber that is then routed past the turbine blades to provide power to the turbine/compressor shaft.  The exhaust from this operation is then expelled through the propelling nozzle.  The nozzle will provide a little more forward force because the exhaust gases are accelerated when passing through the nozzle.  The net force will be in a direction opposite the incoming air flow.   


These gas turbine engines will require atmospheric oxygen for fuel combustion where rocket engines do not.  The rockets carry their own oxygen with them inside.  Both rockets and turbines provide forward thrust in the same manner, however.  Both rely on accelerating mass to provide an equal and opposite force.  Newton’s law never specifies what the mass must be.  In a turbine engine, it’s outside air and combustion products.  On a rocket its all combustion products.  On a ship or a boat water is accelerated by the propellor to provide forward thrust. 
Title: Re: Do rockets push off the air?
Post by: Action80 on December 08, 2023, 06:52:42 PM
You’re holding onto someone in a vacuum.
You push off each other. According to Action80, only one of you should move (the gas should move but not the rocket).[
Two people pushing off each other is actually a force pair. You are almost getting it.
After all, before you both push you’re a “closed system”. It’s almost as if, when you push off the other person, the le momentum is… le conserved.
Trying to equate two people pushing off each other to the operation of a rocket is just plain stupid, so do everyone a favor and stop posting bs.

Rockets are, in fact, observed to gain efficiency at higher altitudes with less air resistance (varying slightly with the engine’s specified job)
No shit. You have any other obvious tidbits of drivel to add to your own op?

Rocket engines can certainly operate at higher altitudes than jets, but that is only because they carry their own oxidizers and require no air intake to accomplish combustion. Once external environment pressure drops below a certain level (i.e., pressures reported at or below the supposed "outer space"), rockets can no longer achieve propulsion due to a lack of a force pair.
Title: Re: Do rockets push off the air?
Post by: RonJ on December 08, 2023, 07:27:15 PM
Inside a rocket's combustion chamber there is the introduction of a mass of fuel at a low relative velocity.  The fuel mass is set on fire.  That releases energy.  One end of the combustion chamber is closed to the product of the combusted fuel.  The other end is open to the outside of the engine.  Since the pressure is lower on the outside, the combusted fuel accelerates out in that direction.  The accelerated fuel mass produces a force equal and opposite to its acceleration vector. 
 
Any pressure on the outside of the rocket engine will inhibit the exhausts acceleration.  Since the force is proportional to the mass acceleration the less external force outside the rocket engine the more force will be produced.  This means that a rocket will be more efficient in a vacuum than in an atmosphere.   
Title: Re: Do rockets push off the air?
Post by: Realestfake on December 08, 2023, 07:30:54 PM
Inside a combustion chamber there is the introduction of a mass of fuel at a low relative velocity.  The fuel mass is set on fire.  That releases energy.  One end of the combustion chamber is closed to the product of the combusted fuel.  The other end is open to the outside of the engine.  Since the pressure is lower on the outside, the combusted fuel accelerates out in that direction.  The accelerated fuel mass produces a force equal and opposite to its acceleration vector. 
 
Any pressure on the outside of the rocket engine will inhibit the exhausts acceleration.  Since the force is proportional to the mass acceleration the less external force outside the rocket engine the more force will be produced.  This means that a rocket will be more efficient in a vacuum than in an atmosphere.

This is correct.


Trying to equate two people pushing off each other to the operation of a rocket is just plain stupid, so do everyone a favor and stop posting bs.
Do explain  :) a person pushes off of you, moving you in the opposite direction. The gas pushes off of the rocket, moving the rocket in the opposite direction. Both examples start as closed systems, and end with two parts separated by the force.
Title: Re: Do rockets push off the air?
Post by: Action80 on December 08, 2023, 07:36:20 PM
Inside a rocket's combustion chamber there is the introduction of a mass of fuel at a low relative velocity.  The fuel mass is set on fire.  That releases energy.  One end of the combustion chamber is closed to the product of the combusted fuel.  The other end is open to the outside of the engine.  Since the pressure is lower on the outside, the combusted fuel accelerates out in that direction.  The accelerated fuel mass produces a force equal and opposite to its acceleration vector. 
 
Any pressure on the outside of the rocket engine will inhibit the exhausts acceleration.  Since the force is proportional to the mass acceleration the less external force outside the rocket engine the more force will be produced.  This means that a rocket will be more efficient in a vacuum than in an atmosphere.
You have absolutely no idea what you are writing. Gas released to vacuum performs 0 work. It freely expands.

Joule's Law.
Title: Re: Do rockets push off the air?
Post by: Action80 on December 08, 2023, 07:40:02 PM
Inside a combustion chamber there is the introduction of a mass of fuel at a low relative velocity.  The fuel mass is set on fire.  That releases energy.  One end of the combustion chamber is closed to the product of the combusted fuel.  The other end is open to the outside of the engine.  Since the pressure is lower on the outside, the combusted fuel accelerates out in that direction.  The accelerated fuel mass produces a force equal and opposite to its acceleration vector. 
 
Any pressure on the outside of the rocket engine will inhibit the exhausts acceleration.  Since the force is proportional to the mass acceleration the less external force outside the rocket engine the more force will be produced.  This means that a rocket will be more efficient in a vacuum than in an atmosphere.

This is correct.

This in incorrect. Gas released to a vacuum performs 0 work.

Joule's Law.


Trying to equate two people pushing off each other to the operation of a rocket is just plain stupid, so do everyone a favor and stop posting bs.
Do explain  :) a person pushes off of you, moving you in the opposite direction. The gas pushes off of the rocket, moving the rocket in the opposite direction. Both examples start as closed systems, and end with two parts separated by the force.
You are clearly stating the gas exhaust (something which is part of the rocket, a single closed system, as something entirely separate, like the other person.

It is foolish and you are writing crapola.

Look, I don't care how many screwed-up alts you want to recruit to chime in.

As long as your alive and post crap like tyou are posting now, you will remain wrong.
Title: Re: Do rockets push off the air?
Post by: Realestfake on December 08, 2023, 07:41:12 PM
You have absolutely no idea what you are writing. Gas released to vacuum performs 0 work. It freely expands.

Joule's Law.

That is literally not what Joule’s Law is. Joule’s Law is about the proportionality of heat generated and current through a conductor. I think we’re about wrapped up here.
Title: Re: Do rockets push off the air?
Post by: Realestfake on December 08, 2023, 07:44:34 PM
You are clearly stating the gas exhaust (something which is part of the rocket, a single closed system, as something entirely separate, like the other person.

It is foolish and you are writing crapola.

You are objectively wrong. Two people attached to each other is a closed system in the same way gas inside a fuel tank is.

A person pushes off of you, moving you in the opposite direction. The gas pushes off of the rocket, moving the rocket in the opposite direction. Both examples start as closed systems, and end with two parts separated by the force. And neither examples have anything to do with atmosphere.
Title: Re: Do rockets push off the air?
Post by: Action80 on December 08, 2023, 07:46:49 PM
You have absolutely no idea what you are writing. Gas released to vacuum performs 0 work. It freely expands.

Joule's Law.

That is literally not what Joule’s Law is. Joule’s Law is about the proportionality of heat generated and current through a conductor. I think we’re about wrapped up here.
Joule's expansion.

It is about gas freely expanding when it is released to a vacuum.

Forms no plume.

You are wrapped alright. Probably in a straight jacket or something.

Title: Re: Do rockets push off the air?
Post by: RonJ on December 08, 2023, 07:48:23 PM
Yes, the gas would freely expand once it was outside of the rocket engine.   At that point the exhaust gas mass would have already been accelerated and an equal and opposite force would be applied to the forward end of the combustion chamber.  The fuel is not being burned in a vacuum but inside the combustion chamber enclosure.  A rocket would be more efficient if the gas was dispersed immediately once it exited the rocket.  The action – reaction part would already be completed. 
Title: Re: Do rockets push off the air?
Post by: Realestfake on December 08, 2023, 07:49:37 PM
Joule's expansion.

It is about gas freely expanding when it is released to a vacuum.

Sorry, but Joule (not Joule’s) expansion is not the same thing as Joule’s Law. Gases expanding in space has nothing to do with the reaction force of the combustion moving the rocket.
Title: Re: Do rockets push off the air?
Post by: Action80 on December 08, 2023, 07:52:04 PM
You are clearly stating the gas exhaust (something which is part of the rocket, a single closed system, as something entirely separate, like the other person.

It is foolish and you are writing crapola.

You are objectively wrong. Two people attached to each other is a closed system in the same way gas inside a fuel tank is.
Trying to claim a pair of people are a single unit...FUCKING HILARIOUS!!!

Tell you what, Copernicus...

Draw a goddamn diagram of a pair of people acting as a single closed system, while at the same fucking time pushing off each other so they can go opposite directions, and submit the diagram for critique to a science professor.

Once he okays that piece of crap, then post it here with the verification and I'll concede, okay?

A person pushes off of you, moving you in the opposite direction. The gas pushes off of the rocket, moving the rocket in the opposite direction. Both examples start as closed systems, and end with two parts separated by the force.
Just more crap that is so goddamn wrong it boggles the mind.

You are truly a piece of work.
Title: Re: Do rockets push off the air?
Post by: Action80 on December 08, 2023, 07:54:51 PM
Joule's expansion.

It is about gas freely expanding when it is released to a vacuum.

Sorry, but Joule (not Joule’s) expansion is not the same thing as Joule’s Law. Gases expanding in space has nothing to do with the reaction force of the combustion moving the rocket.
Yeah, Joule has more than one.

Gas, when released to vacuum, performs 0 work.

It is a law of physics.

It is named after Joule.

Ergo, Joule's Law.

It is the product of the combustion that moves the rocket, you dyngus.
Title: Re: Do rockets push off the air?
Post by: markjo on December 09, 2023, 01:26:30 AM
The exhausted gas is part of the rocket. A rocket is a closed system.
If you want to consider a rocket to be a closed system, then you must not ignore conservation of momentum.  If the exhaust is being accelerated one way, then the rest of the rocket must be accelerated the opposite way in order for momentum to be conserved.  That is, unless you think that accelerating rocket exhaust doesn't exhibit momentum.


Gas, when released to vacuum, performs 0 work.
True, but irrelevant.  All of the relevant work is done inside the rocket engine, before the exhaust is released into the vacuum.
Title: Re: Do rockets push off the air?
Post by: RonJ on December 09, 2023, 01:54:58 AM
Imagine some fuel inside a rocket. That fuel has some mass.  The fuel and the rocket have about the same relative velocity.  Now some fuel is inducted into the combustion chamber and combustion starts and the fuel’s energy is released.  The combustion chamber is closed at one end and open at the exhaust end.  Pressure inside the combustion chamber builds up due to the energy released by burning the fuel.   The combusted fuel’s mass is ejected at an accelerated rate out the exhaust end.  Newton’s law would say the accelerated fuel mass would produce an equal and opposite reaction.  That opposite force vector would be in the general direction of the nose of the rocket.


Here is your ‘force pair’.  The mass of the combusted rocket fuel being accelerated toward the exhaust port at the rear of the rocket and the forward part of the combustion chamber attached to the rocket itself.  The rocket’s mass is being accelerated in one direction and the mass of the accelerated fuel in the opposite direction.  The dividing line is the forward part of the combustion chamber as it divides the accelerating mass of the rocket itself in one direction with the accelerating mass of the combusted fuel in the opposite direction.


Any external air pressure at the exhaust end acts like a small back pressure that will slow down the acceleration rate of the exiting combusted fuel a little and reduce the forward acceleration rate of the rocket.  If the rocket is in a vacuum that back pressure will be close to zero and the burned fuel mass will be accelerated at a greater rate.  Everything takes place inside the rocket and the lack of external air has nothing to do with the fuel being accelerated in one direction and the rocket being accelerated in the opposite direction. 
Title: Re: Do rockets push off the air?
Post by: Action80 on December 09, 2023, 09:50:40 AM
The exhausted gas is part of the rocket. A rocket is a closed system.
If you want to consider a rocket to be a closed system, then you must not ignore conservation of momentum.  If the exhaust is being accelerated one way, then the rest of the rocket must be accelerated the opposite way in order for momentum to be conserved.  That is, unless you think that accelerating rocket exhaust doesn't exhibit momentum.
A rocket is a closed system. I haven't ignored the conservation of momentum. You might want to read the thread before making any more lying comments.

Gas, when released to vacuum, performs 0 work.
True.
Fixed that for you.
All of the relevant work is done inside the rocket engine, before the exhaust is released into the vacuum.
Another cosigner to the foolish idea. You are probably the originator of this heaping pile of crap you are writing here in this thread. Just what is the "relevant work " of a rocket?

It is the exhaust plume (i.e., mass ejected, at an accelerated rate) going in one direction causing the rocket or jet to go in the opposite direction.

Do you wish to continue to display your lies and/or ignorance to the audience, penguin?
Title: Re: Do rockets push off the air?
Post by: Action80 on December 09, 2023, 01:25:08 PM
Imagine some fuel inside a rocket. That fuel has some mass.  The fuel and the rocket have about the same relative velocity.  Now some fuel is inducted into the combustion chamber and combustion starts and the fuel’s energy is released.  The combustion chamber is closed at one end and open at the exhaust end.  Pressure inside the combustion chamber builds up due to the energy released by burning the fuel.   The combusted fuel’s mass is ejected at an accelerated rate out the exhaust end.  Newton’s law would say the accelerated fuel mass would produce an equal and opposite reaction.  That opposite force vector would be in the general direction of the nose of the rocket.


Here is your ‘force pair’.  The mass of the combusted rocket fuel being accelerated toward the exhaust port at the rear of the rocket and the forward part of the combustion chamber attached to the rocket itself.  The rocket’s mass is being accelerated in one direction and the mass of the accelerated fuel in the opposite direction.  The dividing line is the forward part of the combustion chamber as it divides the accelerating mass of the rocket itself in one direction with the accelerating mass of the combusted fuel in the opposite direction.


Any external air pressure at the exhaust end acts like a small back pressure that will slow down the acceleration rate of the exiting combusted fuel a little and reduce the forward acceleration rate of the rocket.  If the rocket is in a vacuum that back pressure will be close to zero and the burned fuel mass will be accelerated at a greater rate.  Everything takes place inside the rocket and the lack of external air has nothing to do with the fuel being accelerated in one direction and the rocket being accelerated in the opposite direction.
The mass of the exhaust is part of the closed system. No force pair can be achieved, as a single closed system cannot pair with itself.
Title: Re: Do rockets push off the air?
Post by: markjo on December 09, 2023, 05:34:01 PM
A rocket is a closed system. I haven't ignored the conservation of momentum.
You do understand that a closed system can have more than one component, don't you?  The rocket engine is one component and the propellant is another.  If the propellant is burnt in the combustion chamber causing the exhaust gasses to be accelerated out the back, then what balances the momentum of that accelerated exhaust gas?


Just what is the "relevant work " of a rocket?

It is the exhaust plume (i.e., mass ejected, at an accelerated rate) going in one direction causing the rocket or jet to go in the opposite direction.
What causes the exhaust plume to accelerate in one direction?  Or do exhaust gasses not require a force pair to accelerate?
Title: Re: Do rockets push off the air?
Post by: Action80 on December 09, 2023, 07:07:49 PM
A rocket is a closed system. I haven't ignored the conservation of momentum.
You do understand that a closed system can have more than one component, don't you?
Relevance? 
The rocket engine is one component and the propellant is another.
Oh, here is the relevance. A statement of something very obvious. Thank you.
If the propellant is burnt in the combustion chamber causing the exhaust gasses to be accelerated out the back, then what balances the momentum of that accelerated exhaust gas?
Exhaust gas goes one direction, rocket goes the other direction.

Jesus, how many times must this be written?
Just what is the "relevant work " of a rocket?

It is the exhaust plume (i.e., mass ejected, at an accelerated rate) going in one direction causing the rocket or jet to go in the opposite direction.
What causes the exhaust plume to accelerate in one direction?  Or do exhaust gasses not require a force pair to accelerate?
Remember the issue you had with the words "monstrous jet of energy..." I do, but let us remind the viewing audience of the rest of asshatted posts now, shall we?
The atmoplane doesn't seem very solid to me.  Wouldn't that "monstrous jet of energy" just push the atmoplane out of its way, especially at higher altitudes where the air is much thinner?
Holy crap! It seems a rocket engine is actually capable of creating a "monstrous jet of energy". Jeez, whoda thunk?

And that "monstrous jet of energy" does indeed displace a significant amount of the atmoplane away, but still the external pressure allows the formation of a plume.   

And yes, a force pair is still required. A closed system cannot form a force pair internal to itself. In the case of both jets and rockets, that force pair can only be formed with an outside environment that has measurable air pressure. The rocket can form a working plume in an environment that has less air pressure than a jet (jets are probably capped at an altitude of of 15 miles or so), probably at around 190,000 ft. or so.
Title: Re: Do rockets push off the air?
Post by: DuncanDoenitz on December 09, 2023, 08:03:50 PM


And yes, a force pair is still required. A closed system cannot form a force pair internal to itself. In the case of both jets and rockets, that force pair can only be formed with an outside environment that has measurable air pressure. The rocket can form a working plume in an environment that has less air pressure than a jet (jets are probably capped at an altitude of of 15 miles or so), probably at around 190,000 ft. or so.


Better check again the definition of a Closed System; it is entirely isolated from its environment. 

A rifle cartridge is, and remains, a closed system; bullet goes one way, case and rifle go the other.  Force pair.  Muzzle blast irrelevant.   

A rocket motor is a closed system; combustion gases go one way, motor goes the other.  Force pair.  Plume irrelevant. 
Title: Re: Do rockets push off the air?
Post by: markjo on December 09, 2023, 08:25:42 PM
A rocket is a closed system. I haven't ignored the conservation of momentum.
You do understand that a closed system can have more than one component, don't you?
Relevance? 
The rocket engine is one component and the propellant is another.
Oh, here is the relevance. A statement of something very obvious. Thank you.
If the propellant is burnt in the combustion chamber causing the exhaust gasses to be accelerated out the back, then what balances the momentum of that accelerated exhaust gas?
Exhaust gas goes one direction, rocket goes the other direction.
So there is a force pair between the rocket and the exhaust gas?  Glad we have that settled.

Jesus, how many times must this be written?
Until we get this next bit straightened out.

A closed system cannot form a force pair internal to itself. In the case of both jets and rockets, that force pair can only be formed with an outside environment that has measurable air pressure.
Okay, I think that I see the source of your confusion.  You don't seem to understand what "closed system" means.  A closed system does not exchange matter with its surroundings.  That's why it's called a CLOSED system. 

You already agreed that closed systems can have more than one component, therefore you must also agree that force pairs can (and indeed, must) exist within a closed system so that momentum is conserved.
Title: Re: Do rockets push off the air?
Post by: Action80 on December 09, 2023, 10:34:26 PM


And yes, a force pair is still required. A closed system cannot form a force pair internal to itself. In the case of both jets and rockets, that force pair can only be formed with an outside environment that has measurable air pressure. The rocket can form a working plume in an environment that has less air pressure than a jet (jets are probably capped at an altitude of of 15 miles or so), probably at around 190,000 ft. or so.


Better check again the definition of a Closed System; it is entirely isolated from its environment. 

A rifle cartridge is, and remains, a closed system; bullet goes one way, case and rifle go the other.  Force pair.  Muzzle blast irrelevant.   

A rocket motor is a closed system; combustion gases go one way, motor goes the other.  Force pair.  Plume irrelevant.
A rocket requires nothing outside of it, unlike a jet, to create thrust, ergo, a closed system.
 
I am laughing at the folly you exhibit claiming the muzzle blast has nothing to do with the recoil on a rifle...just plain bullshit. Can you help yourself at all?

A closed system cannot form a force pair with itself.

Stop posting bullshit.
Title: Re: Do rockets push off the air?
Post by: Action80 on December 09, 2023, 10:46:09 PM

So there is a force pair between the rocket and the exhaust gas?  Glad we have that settled.
The exhaust gas (plume) is part of the rocket. A single system cannot form a force pair with itself.

Okay, I think that I see the source of your confusion.  You don't seem to understand what "closed system" means.  A closed system does not exchange matter with its surroundings.  That's why it's called a CLOSED system. 

You already agreed that closed systems can have more than one component, therefore you must also agree that force pairs can (and indeed, must) exist within a closed system so that momentum is conserved.
I think you are blind to the meaning of the word "exchange."

A rocket does not exchange matter with its surroundings, It only gives matter (i.e., exhaust) to its surroundings. It takes in nothing from its surroundings.

A closed system cannot form a force pair with itself.

The end.
Title: Re: Do rockets push off the air?
Post by: markjo on December 10, 2023, 12:30:27 AM

So there is a force pair between the rocket and the exhaust gas?  Glad we have that settled.
The exhaust gas (plume) is part of the rocket. A single system cannot form a force pair with itself.
If there is no force pair, then what causes the exhaust to accelerate in one direction and the rocket to accelerate in the other?


I think you are blind to the meaning of the word "exchange."

A rocket does not exchange matter with its surroundings, It only gives matter (i.e., exhaust) to its surroundings. It takes in nothing from its surroundings.

A closed system cannot form a force pair with itself.
And you are blind to the meaning of the word "closed". 

Propellant from outside the rocket loaded into it and then forcibly ejected back outside of it.

A closed system does not take in matter from from outside of itself or eject matter to the outside of itself.  That would be an open system.

If you must insist that a rocket is a closed system, then you must understand that the rocket engine and the resulting exhaust gasses are 2 elements within that closed system that can and must force pair because momentum must be conserved in a closed system. 

It isn't a case of the closed system force pairing with itself.  It's a case of the force pair happening within the closed system.  A very significant difference that you don't seem to grasp.
Title: Re: Do rockets push off the air?
Post by: RonJ on December 10, 2023, 03:15:18 AM
If you wish to define the boundaries of a rocket’s system as the entire universe then that rocket could be considered a closed system.  That would be the only way that rocket could be classified as a closed system because the burning fuel exits the rocket’s chassis and travels off to some unknown place after exiting the nozzle. 
 
The conventional way of classifying systems would be to say that mass never enters or leaves a closed system.  Since a conventional rocket has the mass of its fuel leaving out the nozzle on a continuous basis while the rocket is starting its travels, you would conventionally classify it as an open system. 
 
It would be better for you to define your system boundaries to avoid further confusion about whether a rocket is an open or closed system.   
Title: Re: Do rockets push off the air?
Post by: Action80 on December 10, 2023, 06:26:59 AM
If there is no force pair, then what causes the exhaust to accelerate in one direction and the rocket to accelerate in the other?
Have you read the thread?

And you are blind to the meaning of the word "closed". 

Propellant from outside the rocket loaded into it and then forcibly ejected back outside of it.

A closed system does not take in matter from from outside of itself or eject matter to the outside of itself.  That would be an open system.
Losing sight of the word, "exchange".

Have you read the thread?

Or are you just going to continue your lying, gaslighting ways?

If you must insist that a rocket is a closed system, then you must understand that the rocket engine and the resulting exhaust gasses are 2 elements within that closed system that can and must force pair because momentum must be conserved in a closed system. 

It isn't a case of the closed system force pairing with itself.  It's a case of the force pair happening within the closed system.  A very significant difference that you don't seem to grasp.
I see.

You are just going to continue to post bullshit and lie your ass off.
Title: Re: Do rockets push off the air?
Post by: Realestfake on December 10, 2023, 08:10:26 AM
Losing sight of the word, "exchange".

Have you read the thread?

Or are you just going to continue your lying, gaslighting ways?

If you must insist that a rocket is a closed system, then you must understand that the rocket engine and the resulting exhaust gasses are 2 elements within that closed system that can and must force pair because momentum must be conserved in a closed system. 

It isn't a case of the closed system force pairing with itself.  It's a case of the force pair happening within the closed system.  A very significant difference that you don't seem to grasp.
I see.

You are just going to continue to post bullshit and lie your ass off.

Nobody’s lying to or gaslighting you. You’ve wasted half your breath in this thread telling people to “stop posting bs!” “leave reality to the sane!” instead of anything of real substance.
Title: Re: Do rockets push off the air?
Post by: Action80 on December 10, 2023, 10:35:58 AM
Nobody’s lying to or gaslighting you. You’ve wasted half your breath in this thread telling people to “stop posting bs!” “leave reality to the sane!” instead of anything of real substance.
Actually, you are lying and gaslighting me again, right now with ^this post.

I have posted the exact method of how rockets and jets work within this thread and numerous times in various other threads on this forum.

All you have posted is a bunch of lying crap, merely demonstrating your ineptitude and inability to read or understand the subject matter of your op, you faker.
Title: Re: Do rockets push off the air?
Post by: markjo on December 10, 2023, 05:47:09 PM
If there is no force pair, then what causes the exhaust to accelerate in one direction and the rocket to accelerate in the other?
Have you read the thread?
Sorry, but an exhaust plume force pairing with the ever thinning atmoplane doesn't cut it.  That is unless you can show the math that proves that an ever thinning atmpolane can form a force pair that is sufficient to propel several hundred tons of rocket upwards.

Losing sight of the word, "exchange".
Not at all. 
Quote from: http://home.iitk.ac.in/~suller/lectures/lec2.htm
Two types of exchange can occur between system and surroundings: (1) energy exchange (heat, work, friction, radiation, etc.) and (2) matter exchange (movement of molecules across the boundary of the system and surroundings).

You're the one who seems to be losing sight of the word "closed" as it applies to thermodynamic systems.
Quote from: http://home.iitk.ac.in/~suller/lectures/lec2.htm
Based on the types of exchange which take place or don't take place, we will define three types of systems:

    isolated systems: no exchange of matter or energy
    closed systems: no exchange of matter but some exchange of energy
    open systems: exchange of both matter and energy


Have you read the thread?
Have you read a physics text book?


Or are you just going to continue your lying, gaslighting ways?
No lying or gaslighting.  Just trying to educate you.  Some words can have a somewhat different meaning in physics than in everyday usage, so I suppose that I can see how you might not be able to tell the difference.

But if you insist that I'm lying, then please provide a credible source that disagrees with me and agrees with you.  Citing yourself doesn't count.
Title: Re: Do rockets push off the air?
Post by: Action80 on December 10, 2023, 06:00:30 PM
Not at all. 
Quote from: http://home.iitk.ac.in/~suller/lectures/lec2.htm
Two types of exchange can occur between system and surroundings: (1) energy exchange (heat, work, friction, radiation, etc.) and (2) matter exchange (movement of molecules across the boundary of the system and surroundings).

You're the one who seems to be losing sight of the word "closed" as it applies to thermodynamic systems.
Absolutely gaslighting again.

An exchange is where 1 system provides something to a 2nd system in exchange for something the 2nd system provided to system 1.
Have you read a physics text book?
Yes.
No lying or gaslighting.  Just trying to educate you.  Some words can have a somewhat different meaning in physics than in everyday usage, so I suppose that I can see how you might not be able to tell the difference.

But if you insist that I'm lying, then please provide a credible source that disagrees with me and agrees with you.  Citing yourself doesn't count.
You are lying and gaslighting. You cannot even comprehend the sources you provide to support your bankrupt position, so providing additional sources would be foolish. You are claiming that a closed system can form a force pair with itself, ffs!

Gtfo with your bs.
Title: Re: Do rockets push off the air?
Post by: markjo on December 10, 2023, 06:27:28 PM
Absolutely gaslighting again.

An exchange is where 1 system provides something to a 2nd system in exchange for something the 2nd system provided to system 1.
If we were using the word "exchange" in its everyday usage, then you might be correct.  However, we are (or at least I am) using it in the context of its scientific definition, which does not necessarily imply a 2 way trade.

Either way, you're getting hung up on semantics more than the real point: a closed system, by any scientific definition, does not gain or lose matter.  That's why it's called "closed".

You are lying and gaslighting. You cannot even comprehend the sources you provide to support your bankrupt position, so providing additional sources would be foolish.
So I'm just supposed to take your word for it?  ::)

You are claiming that a closed system can form a force pair with itself, ffs!
No, I am not claiming that.  I am claiming that objects within a closed system can, and must, force pair with each other in order to conserve momentum.  I am claiming that the rocket engine and the exhaust are two of the many objects that make up a closed system rocket and form a force pair that allows the exhaust to accelerate in one direction and the rest of the rocket to accelerate in the opposite direction.
Title: Re: Do rockets push off the air?
Post by: Dr Van Nostrand on December 10, 2023, 07:24:15 PM
OK.. Let's try this thought experiment...

There's this dude floating naked in weightless vacuum. He's curled up holding his knees in his arms, floating there, and he has a forceful blast of diarrhea. The particles of fecal matter would give his rectum something to push against with no surrounding atmosphere.
The diarrhea would easily shoot out with forward moment and impinge some momentum to the guy's ass, moving him ever so slightly forward.  In fact, if the blast of diarrhea had enough force and volume, it could act as a directional thruster, like on an orbital satellite.

Even if you want to insist that no movement occurred, imagine that you are in a microscopic submarine riding in the dude's colon when he blasted out the diarrhea. Your tiny sub would become a space ship and ride on a wave of diarrhea at sub-relativistic speeds to reach the next star system in 100,000 years.

This is basic science.
Title: Re: Do rockets push off the air?
Post by: Action80 on December 10, 2023, 08:36:13 PM
If we were using the word "exchange" in its everyday usage, then you might be correct.  However, we are (or at least I am) using it in the context of its scientific definition, which does not necessarily imply a 2 way trade.

Either way, you're getting hung up on semantics more than the real point: a closed system, by any scientific definition, does not gain or lose matter.  That's why it's called "closed".
Exchange means exactly that. Exchange. A closed system can lose matter.
You are lying and gaslighting. You cannot even comprehend the sources you provide to support your bankrupt position, so providing additional sources would be foolish.
So I'm just supposed to take your word for it?  ::)
Doesn't matter to me whether you take my word for it or not.

You have been called out for it by the moderation here at this site, numerous times, being relegated to Purgatory for an extended period for exactly that.

You are claiming that a closed system can form a force pair with itself, ffs!
No, I am not claiming that.  I am claiming that objects within a closed system can, and must, force pair with each other in order to conserve momentum.  I am claiming that the rocket engine and the exhaust are two of the many objects that make up a closed system rocket and form a force pair that allows the exhaust to accelerate in one direction and the rest of the rocket to accelerate in the opposite direction.
BWHAHAHAHA! In other words, "No, I am not claiming that, but let me restate that claim here in direct response."

GTFO...
Title: Re: Do rockets push off the air?
Post by: DuncanDoenitz on December 10, 2023, 11:59:57 PM
@Action80.  A system can comprise several entities.  For example the Joule Experiment, which you sometimes quote, comprises a cylinder, a gas, and (depending on the form used) a piston.  All Markjo is suggesting, and which you apparently find derisible, is that some of these components within the sysem can form a force-pair.  The system as a whole is not "acting against itself", but 2 independant entities within the CLOSED system acting against each other.   

Whilst I'm here, can I ask what you believe the nature of the "plume" to be?  By that, I mean is it a gas, does it have substance, does it have mass?  Would you agree that it is formed from the exhaust material of the rocket motor?  Would you agree that, as it is constantly being generated by the motor, that it must be dispersing at an equal rate?  Where is it going? 

Finally, and this is not part of the debate though it's something another couple of people have mentioned; I've no idea of your education level, nor indeed of any of the correspondents on this thread with the possible exception of RonJ who, like myself, says he is a Licensed Aircraft Maintenance Technician.  What I believe, however, is that everyone debating with you is expressing genuinely held opinions.  Some people may have misunderstood what they have learnt, and everyone seems to have misread what you have written, to the extent that we talk BS, make false claims, lie, strawman and gaslight.  You, on the other hand, are a paragon of truth, despite making repeated claims about the voracity of the "plume" thing, and how it is widely accepted by jet engine manufacturers and your nephew (though entirely without any citations to that effect).  I wonder if the disrespect and aggression shown in your replies is founded in a lack of confidence in your stated opinions. 

Without exception I, and the other correspondents, have treated you with utmost respect.  Like a closed system, it would be nice if this were recipricated. 

Title: Re: Do rockets push off the air?
Post by: markjo on December 11, 2023, 12:59:08 AM
A closed system can lose matter.
No, it cannot.  That's why it's called a closed system.  You're thinking of an open system.  If you don't believe me, then let's ask Professor Google:
 https://letmegooglethat.com/?q=can+a+closed+system+lose+matter

Of course you won't believe the internet either because it uses the word "exchange" in reference to matter entering or leaving a system.  Maybe this video will help:
https://www.youtube.com/watch?v=7RawwstN-Pw


BWHAHAHAHA! In other words, "No, I am not claiming that, but let me restate that claim here in direct response."
*sigh*  Okay, so you don't understand the difference between "with" and "within" either.  Here's a hint: "within" means "inside".  So when I say that objects can force pair within a closed system, I mean that objects can force pair inside a closed system.  Does that help?
Title: Re: Do rockets push off the air?
Post by: Action80 on December 11, 2023, 01:05:55 PM
@Action80.  A system can comprise several entities.  For example the Joule Experiment, which you sometimes quote, comprises a cylinder, a gas, and (depending on the form used) a piston.  All Markjo is suggesting, and which you apparently find derisible, is that some of these components within the sysem can form a force-pair.  The system as a whole is not "acting against itself", but 2 independant entities within the CLOSED system acting against each other.
More cosigning of the gaslighting. Since you guys like this site so much: https://physics.stackexchange.com/questions/419854/why-an-internal-force-cannot-move-a-closed-system-externally

You guys have been claiming over and over again the combusted gas does the work of moving the rocket inside the combustion chamber, which is just pure bullshit.

A closed system cannot form a force with itself.
Whilst I'm here, can I ask what you believe the nature of the "plume" to be?  By that, I mean is it a gas, does it have substance, does it have mass?  Would you agree that it is formed from the exhaust material of the rocket motor?  Would you agree that, as it is constantly being generated by the motor, that it must be dispersing at an equal rate?  Where is it going?
Of course the plume is exhausted gas.  Of course it is being dispersed out the back at a set rate governed by the exhaust nozzle. 

Remember just a day or so ago when you made the asshatted claim that the muzzle blast has nothing to do with the recoil on a rifle, shotgun, etc.? I mean, I cannot believe you made such an obviously stupid claim, but here it is again in all its wonder:
A rifle cartridge is, and remains, a closed system; bullet goes one way, case and rifle go the other.  Force pair.  Muzzle blast irrelevant.   
Aside from being a stupid claim, let's examine what happens when you happen to put a brake on: https://www.silencercentral.com/blog/how-much-does-muzzle-brake-reduce-recoil/#:~:text=A%20muzzle%20brake%20is%20a,the%20muzzle%20behind%20a%20bullet.
If you read that information you will see what happens when you diffuse the escaping gas (i.e., muzzle blast) on the end of the barrel. Recoil is reduced.

When gas is released to vacuum, guess what happens to that gas? Ah, yes...it becomes absolutely diffused! No ability for an opposite reaction to occur.

Finally, and this is not part of the debate though it's something another couple of people have mentioned; I've no idea of your education level, nor indeed of any of the correspondents on this thread with the possible exception of RonJ who, like myself, says he is a Licensed Aircraft Maintenance Technician.  What I believe, however, is that everyone debating with you is expressing genuinely held opinions.
When the opinion is baseless and without merit, I do not care. I will call it bullshit and state very clearly why it is bullshit. 
Some people may have misunderstood what they have learnt, and everyone seems to have misread what you have written, to the extent that we talk BS, make false claims, lie, strawman and gaslight.  You, on the other hand, are a paragon of truth, despite making repeated claims about the voracity of the "plume" thing, and how it is widely accepted by jet engine manufacturers and your nephew (though entirely without any citations to that effect).   

Without exception I, and the other correspondents, have treated you with utmost respect.  Like a closed system, it would be nice if this were recipricated.
I guess "treating others with respect," must entail making false claims about how jets and rockets accomplish motion, and rifles experience recoil.
I wonder if the disrespect and aggression shown in your replies is founded in a lack of confidence in your stated opinions.
Calling out obvious bullshit, lying, and gaslighting is absolutely, totally, 100 percent ! ! !, respectful behavior.

I do not care whether you like it or not.
Title: Re: Do rockets push off the air?
Post by: Action80 on December 11, 2023, 01:17:49 PM
A closed system can lose matter.
No, it cannot.  That's why it's called a closed system.  You're thinking of an open system.  If you don't believe me, then let's ask Professor Google:
 https://letmegooglethat.com/?q=can+a+closed+system+lose+matter

Of course you won't believe the internet either because it uses the word "exchange" in reference to matter entering or leaving a system.  Maybe this video will help:
https://www.youtube.com/watch?v=7RawwstN-Pw
Jesus, you just will not quit posting continued bullshit in response. Amazing! You have not changed one bit.

A rocket is a closed system.

Once it is on the launchpad and ready to fire, it takes in nothing else from any system external to itself. It does, however, lose matter (i.e., its exhaust gas) to its external environment.

No exchange takes place.



BWHAHAHAHA! In other words, "No, I am not claiming that, but let me restate that claim here in direct response."
*sigh*  Okay, so you don't understand the difference between "with" and "within" either.  Here's a hint: "within" means "inside".  So when I say that objects can force pair within a closed system, I mean that objects can force pair inside a closed system.  Does that help?
No....
https://physics.stackexchange.com/questions/419854/why-an-internal-force-cannot-move-a-closed-system-externally
Title: Re: Do rockets push off the air?
Post by: DuncanDoenitz on December 11, 2023, 03:25:00 PM
So, the rifle bullet thing; 

https://www.uu.edu/dept/physics/scienceguys/2002Sept.cfm

Union University uses a machine gun as an example of a closed system. 
Title: Re: Do rockets push off the air?
Post by: RonJ on December 11, 2023, 05:11:22 PM
I consulted my physics book that I used while pursuing my engineering degree years ago.  It seems that many people back in the 1920s also insisted that a rocket wouldn’t work in a vacuum.  It took a person like Robert Goddard to prove that supposition wrong.  There were many in the popular press of the day that had lots of fun and called him “Moon man”.  He finally put a gun inside an evacuated jar that fired a blank and proved without a doubt that a rocket would work in space.  He also earned lots and lots of US Patents for his ideas.  Eventually the press printed some retractions and apologies when the advancing space program proved repeatedly that rockets can and do work in a vacuum. 
 
The bottom line is that either there are countless physics PhDs who are correct about their knowledge of the conservation of momentum, or rockets do not work in space as claimed and the whole think is a big farce perpetrated on the public worldwide. 
Title: Re: Do rockets push off the air?
Post by: Realestfake on December 11, 2023, 06:46:13 PM
In this thread Action80 denied that two people attached to each other is a closed system. He is objectively wrong about what a closed system is in kinematics - this isn’t my opinion. That is a textbook example of simple action/reaction - two people pushing off each other.

Abstractly, it is the same concept as gas pushing against a rocket. Before the action, the two actors (fuel and rocket, person 1 and person 2) are connected. After the action, there is a force that pushes both of them apart from each other. There’s no wriggling out of this being the case.
Sorry Action, but your whole plume obsession is completely unfounded and not based on really anything at all.

There is mountains of research and engineering that goes into the smallest aspect of parts such as RCS thrusters that take payloads to Mars. They are designed in a very particular way to work long-term in the harsh environment of space.


Trying to claim a pair of people are a single unit...FUCKING HILARIOUS!!!

Nice argument. How scientific.

Person is pushed against by person. Rocket is pushed against by exploding fuel. That’s it. Not really worth a 6 page thread.
Title: Re: Do rockets push off the air?
Post by: RonJ on December 11, 2023, 08:00:58 PM
I ‘cheated’ again and fetched my thermodynamics book I used in college and looked at the 2nd chapter where the author did discuss the differences between open and closed systems.  One important matter is the determination of the system boundaries.  If you define your system boundaries as X and there is any mass crossing that boundary then it’s considered to be an OPEN system.  This means that Action’s defined system could be a closed system only if the defined boundaries were the entire universe.  No telling where the substances of the rocket’s plume will end up billions of years from now.  Even if you did assume that the entire universe was the system’s boundaries, you still have the problem of the conservation of momentum.  Fuel inside a rocket is observed to exit at an accelerated rate so to produce that acceleration requires a force.  The burned rocket fuel is observed to be exiting out the back of the rocket so the force cannot be from the outside.  Remember, force and acceleration are both vector quantities.  The accelerated fuel must have had something inside the rocket to push against, and that would be the combustion chamber.  It wouldn’t be surprising to see the rocket move in the direction opposite of the plume assuming that the rocket is free to move and isn’t tied down to something. 
Title: Re: Do rockets push off the air?
Post by: markjo on December 11, 2023, 09:20:34 PM
A rocket is a closed system.

Once it is on the launchpad and ready to fire, it takes in nothing else from any system external to itself. It does, however, lose matter (i.e., its exhaust gas) to its external environment.
If there is truly nothing that anyone can say or do to prove to you that a closed system cannot lose matter to its external environment, then there is no point in further discussion. 
Title: Re: Do rockets push off the air?
Post by: Action80 on December 12, 2023, 03:18:33 AM
I consulted my physics book that I used while pursuing my engineering degree years ago.  It seems that many people back in the 1920s also insisted that a rocket wouldn’t work in a vacuum.  It took a person like Robert Goddard to prove that supposition wrong.  There were many in the popular press of the day that had lots of fun and called him “Moon man”.  He finally put a gun inside an evacuated jar that fired a blank and proved without a doubt that a rocket would work in space.  He also earned lots and lots of US Patents for his ideas.  Eventually the press printed some retractions and apologies when the advancing space program proved repeatedly that rockets can and do work in a vacuum. 
 
The bottom line is that either there are countless physics PhDs who are correct about their knowledge of the conservation of momentum, or rockets do not work in space as claimed and the whole think is a big farce perpetrated on the public worldwide.
Tell you what, find a video where a CO2 canister is opened to a vacuum on the level of that given for outer space (i.e., 1.322 × 10-11 Pa.) If that canister moves, I'll retract.
Title: Re: Do rockets push off the air?
Post by: Action80 on December 12, 2023, 03:26:11 AM
In this thread Action80 denied that two people attached to each other is a closed system.
ITT, faker demonstrates he has no clue what a closed system is.
He is objectively wrong about what a closed system is in kinematics - this isn’t my opinion. That is a textbook example of simple action/reaction - two people pushing off each other.
ITT, faker provides no textbook.
Abstractly, it is the same concept as gas pushing against a rocket. Before the action, the two actors (fuel and rocket, person 1 and person 2) are connected. After the action, there is a force that pushes both of them apart from each other. There’s no wriggling out of this being the case.
Sorry Action, but your whole plume obsession is completely unfounded and not based on really anything at all.
Sorry faker, but the plume of the rocket, which is a closed system.

There is mountains of research and engineering that goes into the smallest aspect of parts such as RCS thrusters that take payloads to Mars. They are designed in a very particular way to work long-term in the harsh environment of space.
Hey, believe your cool story bro. Outer space travel is fiction.


Trying to claim a pair of people are a single unit...FUCKING HILARIOUS!!!

Nice argument. How scientific.

Person is pushed against by person. Rocket is pushed against by exploding fuel. That’s it. Not really worth a 6 page thread.
You wrote a bunch of crap. Two people pushing off of each other is not an example of a closed system, period.

One single person is not a closed system, let alone two people.
Title: Re: Do rockets push off the air?
Post by: Action80 on December 12, 2023, 03:28:12 AM
A rocket is a closed system.

Once it is on the launchpad and ready to fire, it takes in nothing else from any system external to itself. It does, however, lose matter (i.e., its exhaust gas) to its external environment.
If there is truly nothing that anyone can say or do to prove to you that a closed system cannot lose matter to its external environment, then there is no point in further discussion.
If there is truly nothing that anyone can say or do to prove to you that a rocket (i.e., a closed system) can lose matter to its external environment (i.e., exhaust gas) then you need to stfu and never post any more the bs you love so well.
Title: Re: Do rockets push off the air?
Post by: Action80 on December 12, 2023, 03:36:40 AM
So, the rifle bullet thing; 

https://www.uu.edu/dept/physics/scienceguys/2002Sept.cfm

Union University uses a machine gun as an example of a closed system.
Ah, the Union people try to bring up a faulty analogy also. I mean, they do not even place their machine gun on a cart within a vacuum but let us roll with it anyway.

What is being ejected from the end of a machine gun attached to a cart aside from the muzzle blast?

I'll give you three guesses, and the first two do not count.
Title: Re: Do rockets push off the air?
Post by: Realestfake on December 12, 2023, 03:54:36 AM
You wrote a bunch of crap. Two people pushing off of each other is not an example of a closed system, period.

One single person is not a closed system, let alone two people.

You not *getting it* isn’t my problem. I will continue to engage because I believe you’re arguing in good faith.

Quote
Two shopping carts that were left in the parking lot are rolling towards each other. The two shopping carts collide. Do the two shopping carts form an open momentum system or a closed momentum system?

Step 1: Determine if there are any external forces acting on the momentum system.

There are no external forces acting on the system. The two shopping carts apply forces to each other, but nothing outside of the system applies a force.

Step 2: Identify the system as open or closed.

An open system will have external forces acting on it.
A closed system will have no external forces acting on it.
Since the shopping carts do not have any external forces acting on them when they collide, this is an example of a closed momentum system.
https://study.com/skill/learn/identifying-open-closed-momentum-systems-explanation.html

Trying to claim a pair of people are a single unit...FUCKING HILARIOUS!!!
Trying to claim a pair of shopping carts are a single unit...FUCKING HILARIOUS!!!

Literally has nothing to do with atmosphere.
Two shopping carts exert a force on one another. Two people exert a force on one another. Exploding fuel and rocket exert a force on one another.
Title: Re: Do rockets push off the air?
Post by: markjo on December 12, 2023, 04:15:02 AM
If there is truly nothing that anyone can say or do to prove to you that a rocket (i.e., a closed system) can lose matter to its external environment (i.e., exhaust gas) then you need to stfu and never post any more the bs you love so well.
I agree wholeheartedly that a rocket can, and does, lose matter to its external environment.  I simply don't agree that the rocket does so as a closed system. 

If you can cite a credible scientific source that agrees with you that a closed system can lose matter to its external environment, then I will admit that I'm wrong and you were right all along.  If you insist on your interpretation of word "exchange" to mean that any transfer of matter must always be both ways at the same time, then your citation must include that as well.

That's what it takes to change my mind.  What does it take to change yours?
Title: Re: Do rockets push off the air?
Post by: Action80 on December 12, 2023, 04:17:15 AM
You wrote a bunch of crap. Two people pushing off of each other is not an example of a closed system, period.

One single person is not a closed system, let alone two people.

You not *getting it* isn’t my problem. I will continue to engage because I believe you’re arguing in good faith.
If you are going to "engage," then you should argue in good faith.

Writing, "Two people are closed system.", is not an argument in good faith.

It is bullshit, plain, pure, and simple.

Quote
Two shopping carts that were left in the parking lot are rolling towards each other. The two shopping carts collide. Do the two shopping carts form an open momentum system or a closed momentum system?
Irrelevant.

Step 1: Determine if there are any external forces acting on the momentum system.

There are no external forces acting on the system. The two shopping carts apply forces to each other, but nothing outside of the system applies a force.

Step 2: Identify the system as open or closed.

An open system will have external forces acting on it.
A closed system will have no external forces acting on it.
Since the shopping carts do not have any external forces acting on them when they collide, this is an example of a closed momentum system.
https://study.com/skill/learn/identifying-open-closed-momentum-systems-explanation.html
Irrelevant.

Trying to claim a pair of people are a single unit...FUCKING HILARIOUS!!!
Trying to claim a pair of shopping carts are a single unit...FUCKING HILARIOUS!!!

Literally has nothing to do with atmosphere.
Two shopping carts exert a force on one another. Two people exert a force on one another. Exploding fuel and rocket exert a force on one another.
Gaslighting bullshit.
Title: Re: Do rockets push off the air?
Post by: Action80 on December 12, 2023, 04:19:33 AM
If there is truly nothing that anyone can say or do to prove to you that a rocket (i.e., a closed system) can lose matter to its external environment (i.e., exhaust gas) then you need to stfu and never post any more the bs you love so well.
I agree wholeheartedly that a rocket can, and does, lose matter to its external environment.  I simply don't agree that the rocket does so as a closed system. 

If you can cite a credible scientific source that agrees with you that a closed system can lose matter to its external environment, then I will admit that I'm wrong and you were right all along.  If you insist on your interpretation of word "exchange" to mean that any transfer of matter must always be both ways at the same time, then your citation must include that as well.

That's what it takes to change my mind.  What does it take to change yours?
A rocket is a closed system.

It is a fact it loses matter to its external environment.

It takes in nothing in exchange from its external environment.

All facts.
Title: Re: Do rockets push off the air?
Post by: markjo on December 12, 2023, 04:24:14 AM
A rocket is a closed system.

It is a fact it loses matter to its external environment.

It takes in nothing in exchange from its external environment.

All facts.
If that's true, then you shouldn't have any trouble citing a credible source that agrees with you.  If you can't do that, then don't bother replying.
Title: Re: Do rockets push off the air?
Post by: Realestfake on December 12, 2023, 04:39:38 AM
You wrote a bunch of crap. Two people pushing off of each other is not an example of a closed system, period.

One single person is not a closed system, let alone two people.

You not *getting it* isn’t my problem. I will continue to engage because I believe you’re arguing in good faith.

Quote
Two shopping carts that were left in the parking lot are rolling towards each other. The two shopping carts collide. Do the two shopping carts form an open momentum system or a closed momentum system?

Step 1: Determine if there are any external forces acting on the momentum system.

There are no external forces acting on the system. The two shopping carts apply forces to each other, but nothing outside of the system applies a force.

Step 2: Identify the system as open or closed.

An open system will have external forces acting on it.
A closed system will have no external forces acting on it.
Since the shopping carts do not have any external forces acting on them when they collide, this is an example of a closed momentum system.
https://study.com/skill/learn/identifying-open-closed-momentum-systems-explanation.html

Trying to claim a pair of people are a single unit...FUCKING HILARIOUS!!!
Trying to claim a pair of shopping carts are a single unit...FUCKING HILARIOUS!!!

Literally has nothing to do with atmosphere.
Two shopping carts exert a force on one another. Two people exert a force on one another. Exploding fuel and rocket exert a force on one another.

Do explain how it’s “irrelevant” and “gaslighting bullshit”. I was kind of hoping for a more intelligent response, but that kind of non-response is perfectly fine with me.

I included a source describing how two X’s pushing each other is a closed system. Please include a source that states otherwise.
Title: Re: Do rockets push off the air?
Post by: Action80 on December 12, 2023, 06:14:54 AM
A rocket is a closed system.

It is a fact it loses matter to its external environment.

It takes in nothing in exchange from its external environment.

All facts.
If that's true, then you shouldn't have any trouble citing a credible source that agrees with you.  If you can't do that, then don't bother replying.
You have not posted anything that disagrees with it.

There are plenty of videos on this forum conclusively demonstrating rockets ejecting matter into the external environment.

It is established a rocket is a closed system.
Title: Re: Do rockets push off the air?
Post by: Action80 on December 12, 2023, 06:17:01 AM

Do explain how it’s “irrelevant” and “gaslighting bullshit”. I was kind of hoping for a more intelligent response, but that kind of non-response is perfectly fine with me.

I included a source describing how two X’s pushing each other is a closed system. Please include a source that states otherwise.
Your example is irrelevant to the op and not analogous to the operation of a rocket.

It is therefore gaslighting bullshit.
Title: Re: Do rockets push off the air?
Post by: Realestfake on December 12, 2023, 06:42:11 AM
Your example is irrelevant to the op and not analogous to the operation of a rocket.

It is therefore gaslighting bullshit.

Before we go further, you’re dead serious right? Like you actually think this? I showed study material that directly contradicts what you claimed. Two objects exerting a force upon each other is clearly called a closed system.
Two carts pushing on each other is a closed system.
Two people pushing on each other is a closed system.
Exploding fuel and rocket pushing on each other is a closed system.
It is, in fact, analogous. This isn’t my opinion.

I’m not sure what properties of fuel and rockets makes the system immune to conservation of momentum.
Title: Re: Do rockets push off the air?
Post by: Action80 on December 12, 2023, 07:22:35 AM
Your example is irrelevant to the op and not analogous to the operation of a rocket.

It is therefore gaslighting bullshit.

Before we go further, you’re dead serious right? Like you actually think this? I showed study material that directly contradicts what you claimed. Two objects exerting a force upon each other is clearly called a closed system.
The example you provided has nothing whatsoever to do with your very own OP, which is, I remind you, Do rockets push off the air?

So, it doesn't directly contradict me, doesn't involve two people, doesn't involve rockets, and remains gaslighting bullshit.
Two carts pushing on each other is a closed system.
Two people pushing on each other is a closed system.
Exploding fuel and rocket pushing on each other is a closed system.
It is, in fact, analogous. This isn’t my opinion.

I’m not sure what properties of fuel and rockets makes the system immune to conservation of momentum.
I never stated anywhere rockets are immune to conservation of momentum.

For the final fucking time, two people are not a closed system. One person is not a closed system.

Neither people pushing off each other or shopping carts rolling in a fucking parking lot are analogous to the operation of a rocket.

You are simply gaslighting in your own OP.
Title: Re: Do rockets push off the air?
Post by: markjo on December 12, 2023, 12:37:30 PM
A rocket is a closed system.

It is a fact it loses matter to its external environment.

It takes in nothing in exchange from its external environment.

All facts.
If that's true, then you shouldn't have any trouble citing a credible source that agrees with you.  If you can't do that, then don't bother replying.
You have not posted anything that disagrees with it.

There are plenty of videos on this forum conclusively demonstrating rockets ejecting matter into the external environment.

It is established a rocket is a closed system.
You have not cited anything that says that a closed system can eject matter into the external environment.   I have cited several examples saying that closed systems cannot eject matter into the external environment.  I can cite more if you want. Can you cite any?
Title: Re: Do rockets push off the air?
Post by: Realestfake on December 12, 2023, 06:49:54 PM
Neither people pushing off each other or shopping carts rolling in a fucking parking lot are analogous to the operation of a rocket.

You are simply gaslighting in your own OP.

“They’re not analogous” then explain why. They really are, and everyone understands this but you.

A cart pushing off a cart is factually operating on the same principle as a rocket pushing off its propellant. Has nothing to do with plumes. If they’re not operating off the same principle, please provide in-depth reasoning and citations.

Gases push on the inside of the rocket pushing it forward, and are expelled backwards. It’s that simple.
Title: Re: Do rockets push off the air?
Post by: Action80 on December 12, 2023, 11:13:11 PM
A rocket is a closed system.

It is a fact it loses matter to its external environment.

It takes in nothing in exchange from its external environment.

All facts.
If that's true, then you shouldn't have any trouble citing a credible source that agrees with you.  If you can't do that, then don't bother replying.
You have not posted anything that disagrees with it.

There are plenty of videos on this forum conclusively demonstrating rockets ejecting matter into the external environment.

It is established a rocket is a closed system.
You have not cited anything that says that a closed system can eject matter into the external environment. Visual evidence of a rocket exhausting gas is not evidence.

FTFY.
Title: Re: Do rockets push off the air?
Post by: Action80 on December 12, 2023, 11:17:51 PM
Neither people pushing off each other or shopping carts rolling in a fucking parking lot are analogous to the operation of a rocket.

You are simply gaslighting in your own OP.

“They’re not analogous” then explain why. They really are, and everyone understands this but you.

A cart pushing off a cart is factually operating on the same principle as a rocket pushing off its propellant. Has nothing to do with plumes. If they’re not operating off the same principle, please provide in-depth reasoning and citations.

Gases push on the inside of the rocket pushing it forward, and are expelled backwards. It’s that simple.
Jesus, you are really funny.

Compile an op and have 0 clue.

Shopping carts playing "bumper cars", two people are closed system, all of it analogous to the operation of rockets...

Honestly, what you write is so stupid it really is not worth the time.
Title: Re: Do rockets push off the air?
Post by: RonJ on December 13, 2023, 12:48:45 AM
It still looks like there is a problem with your contention that a rocket is a closed system.  I suppose that you can define your own systems anyway you want, but you can see where it could cause some confusion when you deviate from the scientific norms at use by most scientists and engineers.  When I consult my university thermodynamics textbook, I do find that there can be NO matter transfer in or out of a CLOSED system.  Obviously, a rocket has a very massive exhaust as the majority of a large rocket’s mass is fuel and when the fuel is mostly burned the majority of the initial rocket’s mass has left the system.  Of course, you could include the rocket’s plumb as part of the system boundaries but then those boundaries would have to be continuously expanding over time. 
 
It's also interesting that you consider a human to be an OPEN system.  That definition would be agreeable to most.  Obviously, a human takes in mass in the form of food and water and ejects the waste mass that the body doesn’t use into a toilet somewhere.  That's pretty simular to a rocket that must be fueled before departure (something that is also contrary to a closed system) and then ejects most of the fuel's mass to generate thrust. 
https://en.wikipedia.org/wiki/Closed_system (https://en.wikipedia.org/wiki/Closed_system)


The information in the link agrees with my text books so you can count on it being accurate.  Please be more specific, if you can, regarding your unique definition of a CLOSED system and specify its boundaries. 
Title: Re: Do rockets push off the air?
Post by: markjo on December 13, 2023, 01:35:32 AM
Visual evidence of a rocket exhausting gas is not evidence.

FTFY.
*sigh*
Two simple questions for you:
1) Is momentum conserved in a closed system?
2) Is momentum conserved if matter leaves a closed system?